How can you prove that the limit of a sequence is a limit point of a set?

Click For Summary
To prove that the limit of a sequence (an) is a limit point of a set A, one must show that every epsilon neighborhood of the limit point x intersects A at points other than x. Given that x = lim(an), for sufficiently large n, the terms of the sequence (an) will be within the epsilon neighborhood (x - e, x + e). Since (an) is not equal to x for all n, there will be infinitely many terms of the sequence within this neighborhood, confirming that x is a limit point of A. The discussion highlights the importance of understanding the relationship between limit points, neighborhoods, and the behavior of sequences. Overall, the concept of limit points in relation to sequences and neighborhoods is clarified through this exploration.
dancergirlie
Messages
194
Reaction score
0

Homework Statement


Let x \in R, and let A \subset R. Let (an) be a sequence with an \in A and (an) \neq x for all n \in N, and assume
that x = lim (an.) Prove that x is a limit point of A.


Homework Equations





The Attempt at a Solution


Suppose that x=lim(an). Meaning there exists a N \in N so that for n\geqN:
|an-x|<\epsilon

Which would make the \epsilon- neighborhood of (an)= (an- \epsilon, an+ \epsilon)

After this I don't know what to do. I need to show that every epsilon neighborhood of V(x) intersects A in some point other than x.

Any tips would be great!
 
Physics news on Phys.org
Careful, your e-neighborhood is off. Remember |a_n - x| < e means x - e < a_n < x + e or a_n is in (x - e, x + e). Now you know that a_n =/= x for each n, so what can you conclude.

Of course this proposition is an if and only if, but this direction is probably clearer since "all but a finitely many points" certainly implies "at least one".
 
alright so if the epsilon neighborhood is (x-e,x+e) and (an) is unequal to x for all n in N would that mean that for some N in N when n>=N that (an) is contained in the epsilon neighborhood. Which would mean that x is the limit point of (an)?
 
oh wait, nevermind, i think I got it. I was trying to make it more complicated than it really was. I am finding this whole open sets, closed sets and limit points idea kind of confusing.
 
Question: A clock's minute hand has length 4 and its hour hand has length 3. What is the distance between the tips at the moment when it is increasing most rapidly?(Putnam Exam Question) Answer: Making assumption that both the hands moves at constant angular velocities, the answer is ## \sqrt{7} .## But don't you think this assumption is somewhat doubtful and wrong?

Similar threads

  • · Replies 13 ·
Replies
13
Views
4K
  • · Replies 6 ·
Replies
6
Views
2K
  • · Replies 11 ·
Replies
11
Views
3K
  • · Replies 5 ·
Replies
5
Views
2K
  • · Replies 16 ·
Replies
16
Views
3K
  • · Replies 1 ·
Replies
1
Views
2K
  • · Replies 2 ·
Replies
2
Views
2K
Replies
5
Views
2K
  • · Replies 8 ·
Replies
8
Views
3K
  • · Replies 11 ·
Replies
11
Views
2K